1ORTHOPEDIC MCQS OB 20 SHOULDER AND ELBOW
ORTHOPEDIC MCQS OB 20 SHOULDER AND ELBOW 1
1) A 24-year-old avid volleyball player has noted gradual onset of shoulder fatigue and weakness limiting his game. Radiographs done by his primary care physician were normal and he has failed to improve with 6 weeks of physical therapy. Given the MRI image shown in Figure A, this patients physical exam may reveal weakness with which of the following actions?
1. Adduction
2. Internal rotation
3. Abduction and external rotation
4. Abduction
5. External rotation
Corrent answer: 3
The MRI demonstrates of a ganglion cyst within the suprascapular notch, leading to atrophy of both the supraspinatus and infraspinatus. Thus, the patient would have weakness with both abduction and external rotation.
Extrinsic compression or traction on the suprascapular nerve can result in suprascapular neuropathy. Compression of the nerve may occur at two distinct locations: the suprascapular notch and the spinoglenoid notch. Extrinsic compression of the suprascapular nerve by ganglion cysts can occur at the spinoglenoid notch or, less commonly, at the suprascapular notch. These cysts may originate from the transverse scapular ligament, the fibrous tissue of the scapula, or the glenohumeral joint.
Mittal et al. reviewed the literature and found that the formation of ganglionic cysts in the spinoglenoid fossa occurs with cumulative trauma and leads to entrapment neuropathy of the suprascapular nerve and denervation of the infraspinatus muscle.
Romeo et al. reported on various etiologies of suprascapular neuropathy including traction injury at the level of the transverse scapular ligament or the spinoglenoid ligament and direct trauma to the nerve. They noted that sports involving overhead motion, such as tennis, swimming, and weight lifting, may result in traction injury to the suprascapular nerve, leading to dysfunction. They also reported that the onset of weakness can be subtle and must be differentiated from cervical radiculopathy and degenerative disease of the shoulder.
Figure A depicts a T2 coronal MRI of the shoulder with a cyst easily visualized occupying the suprascapular notch. Illustration A is an algorithm for the management of suprascapular neuropathy. Illustration B is a sagittal MRI from the same patient depicting the ganglion cyst within the suprascapular notch once again leading to atrophy of both the supraspinatus and infraspinatus (asterisks).
Incorrect Answers:
Answer 1&2: The suprascapular nerve does not innervate the muscles that function to adduct and internally rotate the shoulder.
Answer 4&5: Because the suprascapular nerve is being compressed at the suprascapular notch, it will affect the function of both the infraspinatus and supraspinatus.
2) Which of the following best describes a Buford complex?
1. Normal anatomic variant characterized by a cord-like MGHL and an absent anterosuperior labrum
2. Normal anatomic variant characterized by a cord-like SGHL and an absent posterosuperior labrum
3. Abnormal arthroscopic finding characterized by a cord-like MGHL and an absent anterosuperior labrum
4. Abnormal arthroscopic finding characterized by a cord-like SGHL and an
absent posterosuperior labrum
5. Normal anatomic variant characterized by a cord-like MGHL and a sublabral foramen at the anterosuperior labrum
Corrent answer: 1
A Buford complex, first described by Buford in 1994, is a normal anatomical variant seen in 1.5% of individuals and consists of a cord-like MGHL and absent anterosuperior labrum complex. The cord-like MGHL should not be repaired down to the glenoid as this will result in decreased postoperative range of motion.
Rao et al demonstrated three distinct variations of the anterosuperior portion of the labrum with sublabral foramen being more prevalent than a Buford complex. These normal variants were in order of decreasing prevalence: (1) sublabral foramen with a cord-like MGHL (8%), (2) sublabral foramen (3%), and (3) Buford complex: an absence of labral tissue at the anterosuperior quadrant with a cord-like MGHL (1.5%).
Steinbeck et al. inspected cadaver shoulders and found an absence of an MGHL occurred in 15% of shoulders and was associated with signs of pathologic instability, either Hill-Sachs or bony Bankart.
Illustration A is an arthroscopic image of a Buford Complex, labeled "cord-like MGHL".
Illustration V is a video that shows a diagnostic arthroscopy with a Buford complex.
3) A 78-year old female sustained a 4-part proximal humerus fracture on her dominant side 2 days ago and undergoes a shoulder hemiarthroplasty. Intraoperatively, the lesser tuberosity reduction was difficult and placed too close to the greater tuberosity, which was anatomic. What post-operative problem is likely to result due to the position of the lesser tuberosity?
1. external rotation deficit
2. internal rotation deficit
3. multi-directional instability
4. forward elevation weakness
5. elbow flexion weakness
Corrent answer: 1
Placing the lesser tuberosity in a more lateral position will increase tension on the subscapularis and likely lead to a deficit in external rotation. Variable outcomes in the prosthetic reconstruction of 4-part humerus fractures often can be attributed to inconsistent and nonanatomic tuberosity placement.
Frankle et al (2001) examined the effects of tuberosity malposition in proximal humeral reconstruction after 4 part fractures and found out that there was significant alteration in external rotation kinematics and torque requirements. Failure to properly position tuberosity fragments in the horizontal plane may result in insurmountable postoperative motion restriction.
4) A 23-year-old professional pitcher complains of posterior shoulder pain. Physical exam is notable for scapular dyskinesis. No intra articular pathology is found on shoulder MRI. Which of the following should be emphasized in the initial stages of rehabilitation?
1. Isometric shoulder exercises
2. Isokinetic shoulder exercises
3. Closed chain shoulder exercises
4. Coordination of scapular motion with trunk and hip movements 5. Axial loading shoulder exercises
Corrent answer: 4
Scapular dyskinesis is an alteration in the normal motion of the scapula during coordinated scapulohumeral movements. It occurs as a sequela of prior shoulder injury, especially injuries disrupting the activation patterns of
scapular stabilizing muscles. Kibler et. al outlined a rehabilitation protocol to treat scapular dyskinesis. The principle is to treat the problem from proximal to distal. The first stage involves attaining full motion of the scapula and coordinating the scapula with trunk and hip motions. Once this has been achieved, the second stage involves strengthening the scapular musculature. As scapular control is attained, exercises are introduced that place emphasis on the shoulder and arm beginning with flexibility and closed-chain strengthening, and eventually working up to sport-specific functions. Progress is determined by functional improvement rather than a strict time table. Kibler et al outline the presentation, evaluation, and treatment for scapular dyskinesis in the JAAOS review article. They specifically discuss the acute, recovery, and maintenance phases of rehabilitation.
5) A patient is scheduled to undergo arthroscopy for a SLAP tear of his shoulder. Based on the sagittal images of the right shoulder MRI shown in Figure A, what additional physical exam finding is the patient likely to display?
1. Weakness in forward elevation
2. Weakness in internal rotation
3. Weakness in external rotation
4. Positive impingement maneuver
5. Scapular winging
Corrent answer: 3
The MRI shows a cyst in the spinoglenoid notch, which is important to differentiate from a cyst in the suprascapular notch.
A suprascapular cyst can impinge upon the suprascapular nerve prior to innervation of the supraspinatus and infraspinatus muscle, leading to weakness in both muscles. Prolonged impingement on the suprascapular nerve by a spinoglenoid cyst can result in atrophy of the infraspinatus muscles. This would show up as weakness in external rotation on exam. These cysts are associated with SLAP lesions and per literature are formed by a one-way valve effect, where synovial fluid can exit the joint into the cyst but not drain spontaneously.
Chen et al describe 3 cases in which preoperative and postoperative EMG's and MRI imaging documented cyst resolution and return of suprascapular nerve function after arthroscopic spinoglenoid cyst excision and labral repair.
Figure A and Illustration A shows a cyst in the spinoglenoid notch, where it may impinge on the suprascapular nerve as it travels around the glenoid and under the spine of the scapula on its way to innervate the infraspinatus muscle. Illustration B shows the anatomic locations of the suprascapular notch and the spinoglenoid notch and the course of the suprascapular nerve.
6) Arthroscopic subacromial decompression with acromioplasty has been shown to yield lower subjective satisfaction scores in patients with which of the following preoperative factors?
1. Dominant arm involvement
2. Males
3. Workers' compensation
4. Smokers
5. Age <60
Corrent answer: 3
The first two referenced articles found that patients with active worker’s compensation claims reported significantly decreased satisfaction postoperatively compared to the rest of the cohort. The most recent article reported similar scores, but longer time to return to work.
7) A 65-year-old right-hand-dominant man reports acute right shoulder pain and inability to lift his arm overhead after a glenohumeral dislocation while skiing 2 weeks ago. Physical exam reveals active forward elevation to 30 degrees and 3/5 external rotation strength, pain with motion, and intact lateral arm sensation. An MRI is contraindicated due to a pacemaker, and therefore an arthrogram is performed and shown in Figure A. What is the most appropriate treatment option?
1. Shoulder hemiarthroplasty
2. Rotator cuff repair
3. Proximal humerus ORIF
4. Total shoulder arthroplasty
5. Sling immobilization
Corrent answer: 2
The clinical presentation is consistent with an acute rotator cuff tear following a shoulder dislocation, so the most appropriate treatment is a rotator cuff repair.
A shoulder dislocation in a patient >40 years-old commonly results in a rotator cuff tear. An arthogram may be helpful to confirm the diagnosis when an MRI is contraindicated. The arthrogram shows extravasation of the dye into the subacromial space with no evidence of arthritis. A rotator cuff tear allows the dye to leak into the subacromial space, whereas in a normal MRI arthrogram the dye is contained within the joint capsule (Illustration A).
Craig et al described the geyser sign (Illustration B), which is when dye from a shoulder arthrogram leaks into the subacromial space as well as into the AC joint. This is indicative of a long-standing full-thickness RCT that has now involved the AC joint.
Jensen et al review the pathogenesis of rotator cuff arthropathy which they define as the end point in the continuum of severe degenerative changes in the glenohumeral joint.
8) A 22-year-old minor league baseball pitcher is being treated for shoulder pain with a focused rehabilitation program. Figures A and B display rehabilitation manuevers that are critical in the treatment of his shoulder pathology. What is the most likely diagnosis in this athlete?
1. Long head of the biceps tendonosis
2. Glenohumeral internal rotation deficit (GIRD)
3. Subscapularis rupture
4. Superior labral anterior posterior (SLAP) tear
5. Bankart lesion
Corrent answer: 2
Figure A shows a sleeper stretch and Figure B shows a prone internal rotation stretch with scapular stabilization which are both forms of posterior capsular stretching. Baseball pitchers often have excessive external rotation and diminished internal rotation on their throwing shoulder. A rehabilitation program that includes posterior capsular stretching is essential for the treatment of GIRD.
According to the review article by Braun et al, GIRD is a posterior shift in the total arc of motion and is thought to be a physiological adaptation of the shoulder joint to throwing. The treatment of loss of internal rotation is stretching of the posterior capsule.
The Level 3 article by Crockett et al reviewed shoulder CT scans and shoulder range of motion in 25 pitchers and 25 non-throwers. The pitcher group demonstrated a significant increase in humeral head retroversion by CT scan, external rotation at 90°, external rotation in the scapular plane, and total range of motion compared to the non-pitchers.
9) A 34-year-old carpenter has left shoulder pain for the past 3 months following a fall from a ladder. Figure A displays a coronal T2 MR image. Which of the following diagnoses most appropriately describes this patient's lesion?
1. Anterior labral periosteal sleeve avulsion (ALPSA)
2. Partial articular surface tendon avulsion (PASTA)
3. Humeral avulsion of the glenohumeral ligament (HAGL)
4. Superior labral anterior to posterior tear (SLAP)
5. Glenolabral articular disruption (GLAD) lesion
Corrent answer: 2
The MR image shown in Figure A demonstrates a partial articular surface tendon avulsion (PASTA) lesion of the supraspinatus. PASTA lesions can be difficult to diagnose and intra-articular contrast can help to delineate the pathology as seen in Illustration A. Disrupted anterior scapular periosteum differentiates a Bankart lesion from its variants where periosteum remains
intact. An ALPSA lesion (Illustration B) is where the labral-ligamentous complex is displaced medially and shifted inferiorly, rolling up on itself underneath intact periosteum. A GLAD lesion (Illustration C) is a tear of the anterior inferior labrum (nondisplaced) with avulsion of the adjacent glenoid cartilage. A HAGL lesion (Illustration D) is where the inferior glenohumeral ligament avulses from the inferior humeral neck. Superior labral tears anterior to posterior to the biceps root are known as SLAP tears (Illustration E) .
Gartsman et al provide Level 4 evidence of 85 shoulders taken to the OR for impingement symptoms and found a partial tear of the rotator cuff. These shoulders were treated with debridement of the tear and arthroscopic subacromial decompression and had greater than 80% success rate.
The study by Snyder et al reviews 31 patients with partial thickness rotator cuff tears that were treated by arthroscopic debridement of the lesion. They concluded that 84% of the patients had satisfactory results.
10) While recent studies have failed to demonstrate a significant clinical difference, proximal biceps tenodesis compared to tenotomy is felt to possibly result in a lower incidence of which of the following?
1. Arm cramping
2. Elbow flexion weakness
3. Elbow stiffness
4. Shoulder weakness
5. Shoulder stiffness
Corrent answer: 1
Concern for cosmetic deformity (“popeye” deformity) and muscle spasm or cramping has been an argument against performing tenotomy in the past. The long head of the biceps tendon has been implicated as a common source of anterior shoulder pain. Surgical options to treat it include biceps tenodesis by various methods and intraarticular biceps tendon release- tenotomy. It is felt by some surgeons that a tenodesis may decrease subjective arm cramping and improve cosmesis. Of note, recent studies have failed to show a significant difference between the two groups.
Osbahr et al retrospectively looked at their patients who had undergone either tenotomy or tenodesis and found non statistically significant differences. The patient were non randomized to treatment groups and therefore selection bias prohibits definitive conclusions.
Frost et al performed a comprehensive review of the literature comparing the outcomes of tenotony and tenodesis, and noted that the studies were predominantly low quality. They concluded that there is a lack of quality evidence to advocate one technique over the other.
11) What is the preferred treatment for a symptomatic acute acromioclavicular separation where there is a 20% increase in the coracoclavicular distance on AP radiograph compared to the opposite uninjured side?
1. Anatomic coracoclavicular ligament reconstruction
2. Acute repair of acromioclavicular capsule
3. Sling followed by early physical therapy
4. Reduction and retrograde pinning of the acromioclavicular joint 5. Distal clavicle excision
Corrent answer: 3
A 20% increase in the coracoclavicular distance on AP radiograph compared to the uninjured side would classify this AC separation as a Type II based on the Rockwood classification. The preferred treatment for a type II AC separation is non-operative with a sling and early physical therapy.
The reference by Mouhsine et al outlines the typically successful outcomes of non-operative treatment of type I and II AC separations. They found good results but many patients (48%) remained symptomatic after 6 years with activity related pain or AC tenderness and 84% of these patients had radiographic evidence of AC degeneration, distal clavicle lysis, or CC ligament ossification.
The reference by Clarke and McCann reviews the diagnosis and treatment of AC separations. They argue Type I and Type II AC separations are treated nonoperatively, while types IV-VI are nearly always treated surgically. The treatment of Type III remains controversial.
Illustration A depicts the different types of AC separations.
12) A 62-year-old female presents with chronic shoulder pain. She denies any recent or remote history of trauma or infection. A radiograph is provided in Figure A. Which of the following is the most common cause of her findings?
1. Diabetes
2. Syphilis
3. Alcoholism
4. Syringomyelia
5. Uremia
Corrent answer: 4
The clinical presentation is consistent with Charcot neuroarthropathy of the shoulder. The most common cause of neuropathic arthropathy of the shoulder is syringomyelia, although chronic alcoholism and diabetes have also been reported.
Workup for neuropathic arthropathy of the shoulder includes magnetic resonance images of the cervical spine, to look for a syrinx of the central cord.
Hatzis et al retrospectively reviewed a series of patients with neuropathic arthropathy of the shoulder. The etiology of the neuropathic condition was most commonly syringomyelia. They reported that the diagnosis is often missed, and inappropriate surgeries were frequently performed for this etiology.
Clayton et al also reviewed Charcot arthropathy of the shoulder and concluded that although rare, diabetes mellitus should always be ruled out as a cause for this disorder. They report that treatment should not be different with diabetes as the etiology.
Figure A shows a proximal humerus with significant destruction and fragmentation, consistent with Charcot neuroarthropathy
Incorrect Answers:
Answer 1: Diabetes is a documented cause of Charcot neuroarthropathy of the
shoulder, but is less common than Syringomyelia.
Answer 2: Syphilis is no longer the most common cause of these findings. Answer 3: Alcoholism is a documented cause of Charcot neuroarthropathy of the shoulder, but is less common than Syringomyelia.
Answer 5: There is no documented association between Charcot neuroarthropathy and uremia.
13) Triceps ruptures are associated with all of the following EXCEPT?
1. Anabolic steroid use
2. Local corticosteroid injection
3. Renal osteodystrophy
4. Fluoroquinolone use
5. Creatine supplement use
Corrent answer: 5
Creatine supplement use has not been associated with triceps rupture. Triceps ruptures have been reported in weightlifters who bench press heavy weight, use anabolic steroids, have a history of local steroid injections to the triceps tendon, patients with renal osteodystrophy, and with use of fluoroquinolone antibiotics.
Rettig reviews the risk factors associated with triceps rupture as described above.
Sollender at al and Stannard et al presented cases of triceps tendon ruptures in weightlifters who used anabolic steroids or had numerous local steroid injections.
Khaliq and Zhanell reviewed fluoroquinolone tendon injury and reported two cases of triceps rupture.
14) A 72-year-old male who underwent right total shoulder arthroplasty 8 months ago is unable to lift his right hand off his back and has weakness with internal rotation. What is the most likely diagnosis?
1. Brachial neuritis
2. Long head of biceps rupture
3. Subscapularis insufficiency
4. Subscapularis nerve palsy
5. Standard postoperative recovery
Corrent answer: 3
The patient is unable to internally rotate and has a positive lift-off test (inability to lift off hand from behind back), all significant for subscapularis insufficiency which may happen after any anterior approach to the shoulder with takedown of the subscapularis. Subscapular insufficiency may occur with failure of tendon repair or permanent changes to the subscapularis muscle. Clinical findings include internal rotation weakness, increased passive external rotation, weakness to belly press, and an abnormal subscapularis lift-off test (Video A demonstrates lift-off test).
Scheibel et al reports on 25 patients (primary and revision) who underwent open shoulder stabilization with an inverted L-shaped tenotomy approach which lead to atrophy and fatty infiltration on MRI resulting in postoperative subscapularis muscle insufficiency.
15) Which of the following is a primary restraint of anterior and posterior humeral translation at the position of a patient's right shoulder as shown in Figure A?
1. Inferior glenohumeral ligament (IGHL)
2. Middle glenohumeral ligament (MGHL)
3. Superior glenohumeral ligament (SGHL)
4. Coracohumeral ligament (CHL)
5. Coracoacromial ligament (CA)
Corrent answer: 2
Figure A shows a shoulder in 45 degrees of abduction and 45 degrees of external rotation. The MGHL restrains anterior and posterior translation in the midrange of abduction. The CHL limits inferior translation and external rotation when then arm is adducted and limits posterior translation when the arm is flexed, adducted, and internal rotation. The SGHL also restrains inferior translation and external rotation of the adducted shoulder. The IGHL has an anterior band that is the primary restraint to anterior translation at 90 degrees of shoulder abduction. It also has a posterior band to limit posterior translation. The CA ligament prevents superior head migration in rotator cuff deficient shoulders.
The study by Kuhn et al was a cadaveric study of 20 shoulder specimens mounted in a testing apparatus to simulate the thrower's late-cocking position. They found that cutting the entire inferior glenohumeral ligament resulted in the greatest increase in external rotation (approximately 10 degrees).
16) Which of the following is considered the primary static restraint to anterior gleno-humeral translation with the arm in 90 degrees of abduction?
1. Shape of the bony articulation
2. Negative intra-articular pressure
3. Superior gleno-humeral ligament complex
4. Middle gleno-humeral ligament complex
5. Inferior gleno-humeral ligament complex
Corrent answer: 5
The geometry of the bony articulation is inherently unstable. The rotator cuff is a dynamic stabilizer and the capsulolabral tissues are considered static stabilizers. With the arm at 90 degrees abduction, the anterior band of the inferior gleno-humeral ligament complex is the primary static stabilizer to anterior translation. The middle (MGHL) resists anterior translation at 45 degrees of abduction. The superior (SGHL) resists inferior translation with the arm at one's side.
O'Brien et al. describe the functional anatomy of the inferior gleno-humeral complex based on a series of cadaveric dissections. They note that its orientation and design support the functional concept of this single structure as an important anterior and posterior stabilizer of the shoulder joint. The Burra paper is a review of acute upper extremity instability in athletes.
17) A 21-year-old male who is training to become a professional mixed martial artist complains of weakness with forward flexion of the right arm. Four months ago, he sustained several blows and kicks to his right upper extremity, torso, and flank during consecutive training sessions. Physical exam shows the deformity shown in Figure A. Which of the following muscles labeled in Figure B is most likely deficient and leading to his symptoms?
1. A
2. B
3. C
4. D
5. E
Corrent answer: 4
The patients presentation is consistent with medial scapular winging, which is caused by a long thoracic nerve palsy and serratus anterior muscle deficiency. The serratus anterior muscle is shown with the letter D in Figure B.
The serratus anterior muscle draws the scapula forward and abducts the scapula. Medial scapular winging is a result of serratus anterior weakness and can be a result of nerve palsy, direct-blow trauma (most likely the case in this patient's presentation), microtrauma-induced strain, fatigue from repetitive tensile use, or muscle inhibition secondary to underlying glenohumeral
pathology.
Kibler et al (2003) notes that most of the abnormalities in scapular motion and position can be treated by physical therapy to relieve symptoms and to reestablish strength and activation patterns.
Kibler et al (2002) tested the inter/intrarater reliabilities of physicians and physical therapists detecting the presence of scapular dyskinesis in 26 videotaped subjects. They concluded that abnormal shoulder motion could be detected after appropriate education.
Incorrect Answers:
Answer 1: A - Trapezius is the antagonist to serratus anterior, and when unopposed causes the scapula to be pulled medially.
Answer 2: B - Teres major
Answer 3: C - Latissimus
Answer 5: E - Infraspinatus
18) A patient develops shoulder dysfunction and is noted to have medial winging of the scapula. If the EMG shows an abnormaility, which nerve is most likely to be involved?
1. Suprascapular
2. Axillary
3. Long thoracic
4. Thoracodorsal
5. Radial
Corrent answer: 3
Medial scapular winging is usually due to loss of serratus anterior function due to long thoracic nerve palsy.
Injury to the long thoracic nerve can occur during repetitive trauma, penetrating trauma, surgery, prolonged pressure due to positioning, or inflammatory processes. This results in superior elevation and medial translation of the scapula, with medial rotation of the inferior pole due to loss of serratus anterior muscle function. The patient will develop pain due to compensation by other periscapular muscles, with impaired arm elevation. The diagnosis of long thoracic nerve can be confirmed with EMG, with serial examinations every 3 months to follow recovery, which occurs in most cases within 1 year depending on etiology. For those patients with symptomatic
serratus winging for longer than 1 year with EMG evidence of denervation, surgical options such as scapulothoracic fusion, fascial sling suspension, or muscle transfer can be considered.
Kuhn et review different causes of scapular winging. They classify the condition as primary, secondary, or voluntary. Primary scapular winging may be due to neurologic injury, pathologic changes in the bone, or periscapular soft-tissue abnormalities. Secondary scapular winging occurs as a result of glenohumeral and subacromial conditions and resolves after the primary pathologic condition has been addressed. Voluntary scapular winging is not caused by an anatomic disorder and may be associated with underlying psychological issues.
19) A 47-year-old male with a history of a Putti-Platt procedure 20 years ago presents with right shoulder pain with decreased range-of motion. Radiograph is shown in Figure A. What is the most accurate diagnosis?
1. Primary osteoarthritis
2. Post-capsulorrhaphy arthropathy
3. Post-traumatic arthritis
4. Arthritis from poor placement of coracoid transfer
5. Avascular necrosis
Corrent answer: 2
With a history of a Putti-Platt procedure with the radiograph, the patient most likely has post-capsulorrhaphy arthropathy.
Post-stabilization procedure arthritis is thought to occur due to changes in
contact loading in the shoulder joint due to fixing the joint in an incongruent posistion. It can be severe and debilitating, and lead to arthroplasty as a salvage procedure. The Putti-Platt procedure involves a division of the subscapularis tendon and anterior capsule, and realignment of the lateral tendon stump and capsule sewn into the anterior glenoid neck capsular insertion. The "pants-over-vest" style of repair is then finished by sewing the medial tendon stump into the tuberosity, so that external rotation is significantly limited by the soft tissue imbrication. There is no coracoid transfer for this stabilization procedure.
Bigliani et al. reported on a series of similar patients who developed arthritis following surgery for recurrent glenohumeral dislocation. Authors have theorized that instability repair may excessively tighten the joint in one direction and cause a fixed subluxation in the direction opposite from the side of repair, leading to severe degenerative arthritis due to inappropriate contact loading. 77% of patients following arthroplasty after post-capsulorrhaphy arthropathy had an excellent or satisfactory outcome, with improved pain and range of motion.
Figure A demonstrates severe osteoarthrosis of the affected shoulder, with significant joint space narrowing, periarticular osteophyte formation, and subchondral sclerosis.
Incorrect Answers:
Answers 1, 3, 4, 5: While all of these choices can lead to end-stage arthritis and indications for shoulder arthroplasty, the history given in this question stem indicates Answer 2 as the underlying diagnosis. Of course, the capsulorrhaphy may have been performed for traumatic dislocation, but the best answer choice for this stem is Answer 2.
20) A 25-year-old volleyball player has recurrent right shoulder pain. On exam she has right shoulder weakness to external rotation with her arm at her side and atrophy below the scapular spine. There is no external rotation lag sign. Jobe drop arm and hornblower's tests are negative. The O'Brien's active compression test is positive. What will most likely be found on MRI of her shoulder?
1. Partial articular sided tear of the infraspinatus
2. Partial articular sided tear of the supraspinatus
3. Full thickness tear of the infraspinatus
4. Inferior labral tear with quadrangular space cyst
5. SLAP tear and spinoglenoid notch cyst
Corrent answer: 5
This is a classic case of suprascapular nerve impingment at the spinoglenoid notch, likely from a cyst associated with a posterior SLAP tear. Compression of the suprascapular nerve at the spinoglenoid notch causes isolated infraspinatus weakness while compression at the suprascapular notch would affect both the supraspinatus and infraspinatus. A posterior SLAP tear is suspected with a positive O'Brien's active compression test. The labral tear can allow cyst development in the spinoglenoid notch. Jobe drop arm test is abnormal with supraspinatus weakness and hornblower's test is abnormal with teres minor weakness.
Thompson et al first described the findings of suprascapular nerve entrapment by a spinoglenoid notch cyst.
Fehrman et al confirmed six cases of these cysts associated with posterior capsulolabral injuries.
The article by Piasecki et al reviews the diagnosis and treatment of suprascapular neuropathy.
Illustration A shows a typical appearance of a spinoglenoid cyst on MRI. Illustration B demonstrates O'Brien's active compression test whereby resisted arm flexion with the arm in internal rotation causes pain and external rotation relieves pain.
21) A 42-year-old male has a suspected distal biceps rupture with a tendon that can be palpated but is painful during the hook test examination. Which of the following is the most appropriate next step?
1. Operative exploration of distal biceps tendon
2. Immobilization for three weeks followed by repeat physical examination 3. Early physical therapy with emphasis on ROM and strengthening 4. CT scan
5. MRI scan
Corrent answer: 5
It is important to distinguish between complete and partial tears as it guides treatment decisions. Classic physical exam findings of complete tears include: antecubital pain and ecchymosis, non-palpable distal biceps tendon (abnormal hook test), proximal retraction of the biceps muscle, and weakness with supination and flexion. A partial tear often has a normal hook test but has pain with the examination. An MRI is most appropriate for confirmation of a partial
distal biceps rupture, while an MRI is not always required for a complete tear if the exam is conclusive.
The reference by Vardakas et al reports a series of patients initially treated with conservative management for their partial biceps tendon tears. They were all then treated with operative fixation secondary to recalcitrant pain. They note significant improvement in pain at an average of 31 months in all 7 patients without any complications noted.
22) A 50-year-old wheelchair-bound male with a history of traumatic spinal cord injury presents with 6 months of progressive, painless left shoulder weakness and decreased range of motion. He is afebrile and
CBC, ESR, and C-reactive protein levels are normal. A radiograph is shown in Figure A. Early management should include:
1. HIV testing
2. cervical spine MRI
3. repeat ESR, C-reactive protein, CBC
4. emergent open reduction and internal fixation
5. emergent irrigation and drainage
Corrent answer: 2
This patient has a history of spinal cord injury and presents with an upper extremity neuropathic arthropathy, so a syrinx is highly suspected. Figure A demonstrates a Charcot left shoulder.
Hatzis et al demonstrated that of 6 patients with Charcot shoulder, 5 of the 6
patients were found to have syrinx on MRI of the spine as the underlying cause. Therefore, all patients with shoulder neuropathic arthropathy should receive an MRI of the cervical spine.
As discussed by Guille et al., other causes of neuroarthropathy of the shoulder include Chiari malformation, syphilis and diabetes. They reported on a rare case of Charcot shoulder neuroarthropathy from familial sensory neuropathy.
23) When performing an arthroscopic distal clavicle excision for acromioclavicular joint arthrosis, which of the following structures must be preserved to prevent post-operative anteroposterior instability of the clavicle?
1. Trapezoid ligament
2. Anterior and inferior acromioclavicular joint capsule
3. Superior and posterior acromioclavicular joint capsule
4. Coracohumeral ligament
5. Conoid ligament
Corrent answer: 3
Numerous biomechanical studies have shown that the primary restraint to anteroposterior translation of the clavicle is the ligamentous thickenings of the acromioclavicular joint capsule. Debski et al showed in one such study that the strongest of these ligaments is the superior one, verifying the findings of several other authors. They reported that the superior ligament supplies around 50% of the strength against anteroposterior translation, and it is thickest in its posterior aspect. Additionally, the posterior AC ligament adds an additional 25% of the overall strength. For this reason, these ligaments should be preserved when performing a distal clavicle resection. The length of distal clavicle that can be taken and still preserve stability of the joint is highly debated in the literature. The conoid and trapezoid ligaments are the primary restraints to vertical translation at the AC joint. Renfree and Wright review the published anatomic findings around the AC and SC joints, and come to similar conclusions as above.
24) A 32-year-old overhead athlete catches himself with his right hand while slipping on ice and injures his right shoulder. He fails to improve
with therapy, anti-inflammatory medicines, and rest. His MRI is demonstrated in Figure A. What is the most likely diagnosis?
1. HAGL
2. SLAP tear
3. ALPSA
4. Bankart
5. Loose body
Corrent answer: 2
The T2 MRI image demonstrates a superior labral anterior to posterior tear (SLAP tear), and also shows a partial articular sided tear of the supraspinatus tendon. The gold standard for diagnosis of a SLAP tear is arthroscopy. A study by Yoneda et al. demonstrated that the presence of a linear, high-to intermediate intensity area between the superior labrum and the glenoid rim on oblique coronal T2-weighted images had a sensitivity of 41%, a specificity of 86%, and an accuracy of 63% for diagnosing a SLAP tear. However, there have been recent concerns about the overdiagnosis and SLAP repairs, based on increased number of SLAP repairs in ABOS part II case logs, as presented at the AANA 2010 meeting. A HAGL lesion is a humeral avulsion of the anterior inferior glenohumeral ligament (Image in Illustration A). An ALPSA lesion is seen on an axial image and is an anterior labral periosteal sleeve avulsion characterized by the capsulolabral-periosteal complex peeling off the glenoid face and displacing medially on the glenoid neck. A Bankart lesion is an anterioinferior labral tear often caused by an anterior shoulder dislocation.
25) Resection of the coracoacromial ligament during shoulder arthroscopy results in which of the following?
1. Increased glenohumeral joint translation
2. Increased passive shoulder internal rotation
3. Increased axillary recess volume
4. Decreased acromioclavicular joint reactive forces
5. Decreased resting tension in the long head of the biceps
Corrent answer: 1
The CA ligament provides a suspension function and restrains anterior translation through an interaction with the coracohumeral ligament. Resection of the coracoacromial ligament results in increased glenohumeral joint translation.
Lee et al, in a cadaveric study, showed that at 0 degrees and 30 degrees of abduction, release of the coracoacromial (CA) ligament resulted in a significant increase in glenohumeral joint translations, in both the anterior and inferior directions. The authors state that caution should be exercised in the release of the coracoacromial ligament in those with rotator cuff pain associated with glenohumeral instability. This is especially crucial in patients who may later undergo shoulder arthroplasty, as anterior escape leads to significantly worse clinical outcomes.
Illustration A shows the anatomy of the region, including the CA ligament. Illustration B is a radiograph showing anterosuperior escape after CA ligament resection.
26) An 18-year-old high school volleyball player is being treated for multidirectional instability of the right shoulder with a physical therapy program. She has intermittent pain and instability and episodic numbness and weakness in the ipsilateral hand. All of the following are characteristic features of a generalized connective tissue disorder EXCEPT:
1. Elbow hyperextension of the left arm
2. Left 5th finger passive extension beyond 90°
3. Genu recurvatum of the bilateral knees
4. Excessive supination of the left forearm
5. Abducted thumb to reach the ipsilateral forearm (thumb-to-forearm test) of the right hand
Corrent answer: 4
Excessive supination of the left arm is not listed as part of the Beighton 9- point scoring system for hypermobility. All of the other options are part of this scoring system, and a score of >6 is associated with connective tissue disorders such as Marfan's and Ehlers-Danlos Syndrome.
The Level 5 review article by Schenk and Brems notes that generalized ligamentous laxity has been reported in 45% to 75% of patients who have undergone surgery for multidirectional (MDI) shoulder instability. Patients with MDI have pathologic laxity of the glenohumeral joint in more than one direction with at least one of those being inferior. The onset of symptoms is frequently atraumatic, and the chief complaint is often pain more than instability. Patients can experience concomitant recurrent, transient episodes of numbness, tingling, and weakness in the affected extremity. Most patients can be successfully treated nonoperatively with a specific exercise program. If a 6-month trial of nonoperative management fails, then surgical intervention with an inferior capsular shift can be performed.
27) A college baseball pitcher has posterior-superior and anterior pain in his throwing shoulder. On exam, he has a 30 degree loss of internal rotation on the affected side and a positive O'Brien's test. Radiographs
and MRI are normal. While all of the following may be helpful, which of the following exercises should be emphasized most in this patient's rehabilitation program?
1. Sleeper stretches, cross-body stretches, periscapular strengthening 2. Sleeper stretches and subscapularis stengthening
3. External rotation stretches with cuff strengthening
4. External rotation stretches and periscapular strengthening 5. Altering his arm slot and improving pitching mechanics
Corrent answer: 1
The clinical presentation is consistent with GIRD which is treated with aggressive rehabilitation consisting of posterior capsular and cuff stretching.
GIRD (glenohumeral internal rotation deficit) is now commonly recognized in throwing shoulders. Posterior cuff and capsular tightness can cause decreased internal rotation which may cause pain and is implicated in SLAP and articular sided rotator cuff tears. Radiographs and MRI are often normal.
Kibler et al reviews scapular dyskinesis and its relation to shoulder pain. They report treatment of scapular dyskinesis is directed at managing underlying causes and restoring normal scapular muscle activation patterns by kinetic chain-based rehabilitation protocols.
Burkhart et al developed the acronym "SICK" to refer to the findings one sees in this syndrome (Scapular malposition, Inferior medial border prominence, Coracoid pain and malposition, and dysKinesis of scapular movement). This overuse muscular fatigue syndrome is yet another cause of shoulder pain in the throwing athlete.
28) A patient presents complaining of right shoulder pain and weakness following a neck exploration surgery. On exam, he is noted to have winging of the scapula. His EMG shows denervation of the trapezius muscle. This condition is best described as:
1. Lateral winging due to spinal accesory nerve injury
2. Medial winging due to spinal accesory nerve injury
3. Lateral winging due to long thoracic nerve injury
4. Medial winging due to long thoracic nerve injury
5. Scapular dyskinesia due to cervical radiculopathy
Corrent answer: 1
The clinical presentation is consistent with lateral scapular winging due to iatrogenic injury to the spinal accessory nerve.
Scapular winging is a rare debilitating condition that leads to limited functional activity of the upper extremity. Causes include traumatic, iatrogenic, and idiopathic processes that most often result in nerve injury and paralysis of either the serratus anterior, trapezius, or rhomboid muscles. Serratus anterior paralysis, such as from the long thoracic nerve, results in medial winging of the scapula. This is in contrast to the lateral winging generated by trapezius and rhomboid paralysis. Most cases of serratus anterior paralysis
spontaneously resolve within 24 months, while conservative treatment of trapezius paralysis is less effective.
The review by Kuhn et al. classifies scapular winging as primary, secondary, or voluntary. Primary scapular winging may be due to neurologic injury, pathologic changes in the bone, or periscapular soft-tissue abnormalities. Secondary scapular winging occurs as a result of glenohumeral and subacromial conditions and resolves after the primary pathologic condition has been addressed.
Romero et al described the Eden-Lange procedure with lateral transfer of the levator scapulae and rhomboid muscles which can be helpful for lateral winging. They report satisfactory long-term results for the treatment of isolated paralysis of trapezius, but in the presence of an additional serratus anterior palsy or weak rhomboid muscles, the procedure is less successful in restoring shoulder function.
Levy et al describe a clinical forward elevation lag sign for trapezius palsy with resulting "Triangle sign" in the prone position which differentiates this from serratus winging.
29) Your partner performs distal clavicle excisions through an open approach while you prefer to use an arthroscopic approach. He notes that the literature shows both techniques have similar results with the exception of which of the following benefits of an arthroscopic approach?
1. Ability to evaluate the glenohumeral joint
2. Preservation of the coracoclavicular ligaments
3. Preservation of the inferior acromioclavicular ligaments
4. Lower complication rate
5. Decreased surgical time
Corrent answer: 1
The arthroscopic approach with an initial diagnostic arthroscopy of the glenohumeral space prior to subacromial space is felt to be helpful in confirming the diagnosis and identifying other pathology.
Berg et al. reviewed failures of open distal clavicle excision and found that 15 patients had a missed SLAP lesion. Fewer complications, lower infection rate, and decreased surgical time have not been documented in the literature.
Lemos & Tolo reviewed complications resulting from treatment of AC joint pathology. The open technique can often miss other underlying pathology, such as rotator cuff and labral pathology, that may be associated with degenerative changes of the acromioclavicular joint. Therefore, they recommend that even if an open distal clavicle excision is the treatment of choice, an arthroscopic evaluation of the glenohumeral joint be performed at the same time.
30) A 60-year-old man has chronic shoulder pain and weakness. Radiographs show moderate glenohumeral arthritis and narrowing of the acromio-humeral distance. He is scheduled to undergo either hemiarthroplasty or total shoulder arthroplasty. His postoperative function will be most affected by which of the following factors?
1. The integrity of the rotator cuff
2. The integrity of the coracoacromial ligament
3. The presence of glenoid wear
4. The presence of an inferior head osteophyte
5. The extent of AC joint arthritis
Corrent answer: 1
With conventional arthroplasty, the functional outcomes are dependent on the integrity of the rotator cuff. Narrowing of the acromio-humeral distance indicates superior migration of the humeral head which is often seen in cases of rotator cuff deficiency. Further imaging studies such as an MRI or CT arthrogram may be indicated to evaluate the status of the rotator cuff. If there is an irrepairable rotator cuff tear, total shoulder arthroplasty is contra indicated. In that case, the treatment options include hemiarthroplasty (with limited functional outcomes) or more recently, reverse total shoulder arthroplasty.
Ianotti et al. evaluated the influence of several preoperative factors on the outcome of total shoulder arthroplasty and hemiarthroplasty. They performed concomitant rotator cuff repairs in 10% of the patients and found that repairable rotator cuff tears did not negatively affect the outcomes.
Hettrich et al. reviewed the results of a large series of shoulder arthroplasty without glenoid resurfacing. They found the least functional improvements in those patients with rotator cuff arthropathy.
31) A 19-year-old college baseball player has posterior elbow pain despite non-operative treatment for 9 months. He developed acute worsening of pain and inability to throw. His imaging is shown in Figure A. What is the next most appropriate step in management?
1. Elbow arthroscopy
2. Open removal of osteophytes
3. Ulnar nerve transposition
4. Internal fixation with a compression screw
5. Cast immobolization, followed by gradual return to strengthening program Corrent answer: 4
Athletes may develop an olecranon stress fracture as seen in Figure A. Initially, this is treated with rest from activity. If it does not go onto union, ORIF with a compression screw is indicated. This injury is commonly seen in baseball pitchers as well as other higher-level athletes and is thought of as an overuse-type injury. This is part of the valgus-extension overload syndrome. Before it displaces, most of them are treated with a single large screw. Once it is displaced, it can be treated as any olecranon fx is treated (screw +/- tension band, or plate fixation).
The referenced study by Rettig et al is a case series of five pitchers who underwent ORIF with a 7.0mm screw for this pathology. All went onto successful union at a mean of 15.4 weeks with return to throwing at a mean of 29.4 weeks.
Illustration A depicts a compression screw and tension band construct with healed olecranon stress fracture.
32) Which of the following patient scenarios is most appropriate for reverse total shoulder arthroplasty?
1. A 40-year-old laborer severe glenohumeral arthritis and irrepairable rotator cuff tear.
2. A 40-year-old with a painful proximal humerus malunion. 3. A 75-year-old woman with severe arthritis and active overhead motion. 4. A 75-year-old man with painful arthritis and a massive irrepairable rotator cuff tear
5. Failed hemiarthroplasty due to significant glenoid wear.
Corrent answer: 4
The indications for reverse total shoulder arthroplasty includes symptomatic arthritis in an elderly person with irreparable cuff tear with evidence of arthropathy. The patient must have a functioning deltoid. There is concern for glenoid component loosening in younger, more active patients.
Drake et al found "in short-term followup the RTSA relieves symptoms and restores function for patients with cuff tear arthropathy and irreparable rotator cuff tears with pseudoparalysis (preserved deltoid contraction but loss of active elevation). Severely impaired deltoid function, an isolated supraspinatus tear, and the presence of full active shoulder elevation with a massive rotator cuff tear and arthritis are contraindications to RTSA".
Goutallier describes good results for a reverse TSA in patients with a superiorly "migrated humeral head and abnormal cuff function".
33) The superior glenohumeral ligament is under the greatest stress when the humeral head and arm are in which of the following positions?
1. Anteriorly translated with the arm in 90 degrees of abduction and externally rotated
2. Inferiorly translated with the arm in 5 degrees of adduction 3. Anteriorly translated with the arm in 90 degrees of abduction and internally rotated
4. Inferiorly translated with the arm in 45 degrees of abduction and internal rotation
5. Inferiorly translated with the arm in 90 degrees of abduction and neutral rotation
Corrent answer: 2
The role of each glenohumeral ligament has been clearly defined by previous cadaveric studies that have sectioned different ligaments during different periods of stress on the glenohumeral joint. These studies have demonstrated that the superior glenohumeral ligament provides the most restraint to the shoulder joint when the arm is at zero degrees of abduction or in adduction and pulled inferiorly.
Warner et al. tested 11 cadavers with varying amounts of abduction and rotation to see what ligaments provided specific, directional stability to the shoulder joint. They found that the anterior and posterior bands of the inferior glenohumeral ligament provided the most restraint when the arm was abducted. In addition, they found the superior glenohumeral ligament provided the most restraint when the arm was at zero degrees of abduction and pulled inferiorly.
Illustration A shows an anatomic representation of the glenohumeral ligaments.
Incorrect answers:
1: The anterior inferior glenohumeral ligament is stressed when the arm is in this position
3: An arm positioned as such does not classically stress any of the glenohumeral ligaments
4 and 5: The arm is more adducted in answer 2
34) A 35-year-old non-athlete sustains the injury shown in Figure A. An axillary radiograph is also obtained which is normal. Which of the following outcomes has been shown to be associated with reduction and stabilization?
1. Improved patient reported outcomes at 3 months
2. Return to pre-injury sporting activity within 1 year
3. Improved shoulder range of motion (ROM)
4. Improved strength and endurance
5. Improved cosmesis
Corrent answer: 5
The treatment for type III acromioclavicular (AC) joint injuries in non-athletes remains controversial. Multiple studies have demonstrated that with the exception of improved cosmesis with operative treatment, there is no
difference in functional and patient-reported outcomes between nonoperative and operative management.
Type III AC injuries are those with complete rupture of the AC and coracoclavicular (CC) ligaments. Historically, nonoperative management of this type has yielded good results. It is currently debatable whether anatomical stabilization of AC joint dislocation benefits shoulder function. Stabilization techniques include CC screw fixation, coracoacromial ligament transfer, and myriad methods of CC ligament reconstruction.
Murray et al. published a prospective, randomized, controlled trial comparing patient outcomes following nonoperative care versus fixation for high-grade AC injuries, including types III and V. They reported that while the mean degree of radiographic displacement was significantly less in patients following fixation, there was increased DASH scores in favor of nonoperative treatment and no difference in the mean Oxford Shoulder Score between the 2 cohorts. They concluded that surgical fixation confers no functional benefit over nonoperative treatment at 1 year following high-grade disruptions of the AC joint.
Lemos reviewed the evaluation and treatment of the injured AC joint in athletes. He reported that the management of type III AC injuries remains controversial, with many studies demonstrating nonoperative treatment as equal, if not superior to operative treatment. He recommended that type III injuries be treated symptomatically and only operated on when nonsurgical treatment fails.
Figure A depicts a type III AC injury, defined by a torn AC and CC ligaments, with increased CC distance between 25%-100%. Illustration A depicts the Rockwood classification for AC injuries.
Incorrect Answers:
Answer 1: While some studies have definitively demonstrated improved patient-reported outcomes at 3 months with nonoperative management, this has not been the case with operative management.
Answer 2: Return to sport at 1 year has not been shown to be significantly different between nonoperative and operative treatment.
Answers 3 and 4: Improved shoulder ROM, strength, and endurance has not been shown to be superior with operative management.
35) Figure A exhibits arthroscopic images during posterior
debridement of an overhead athlete. Excessive resection and removing normal olecranon most likely will lead to what pathology when returning to play?
1. Medial epicondylitis
2. Valgus elbow instability
3. Lateral epicondylitis
4. Posterolateral rotatory instablity
5. Olecranon stress fracture
Corrent answer: 2
Excessive resection of the posteromedial olecranon can lead to an valgus instability due to iatrogenic medial ulnar collateral ligament laxity due to repetitive stress.
Valgus extension overload and posteromedial impingement typically develop from the repetitive stress placed on the elbow seen in throwers. This is most commonly associated with medial-sided laxity and results in the formation of posteromedial olecranon osteophytes due to the mechanical abutment of the olecranon against the humerus. When symptomatic, these osteophytes may be
treated with either open or arthroscopic excision. However removal of excessive bone may compromise the biomechanical stability of the medial collateral ligament, leading to laxity and pain with valgus stress testing.
Kamineni et al. performed a biomechanical study and demonstrated a stepwise increase in valgus angulation after resecting the posteromedial olecranon in increments of 3mm (from 0-9 mm). The authors recommend that bone removal from the olecranon should be limited only to osteophytes without the removal of normal bone.
Ahmad et al. describe the valgus extension overload phenomenon upon return to play, noting that surgical treatment should be limited to the resection of osteophytes only; normal olecranon should not be resected in order to avoid valgus instability.
Illustration A and B are radiographs showing formation of posteromedial olecranon osteophytes (white arrows).
Incorrect Answers:
Answer 1,3,4 and 5: None of these injury patterns are associated with excessive resection of the posteromedial olecranon and resultant valgus instability.
36) A 34-year-old competitive weightlifter presents with increasing pain during bench pressing. Despite modifications in his workout, he is unable to compete. His physical exam demonstrates weakness in external rotation. Radiographs are unremarkable. His MRI findings are seen in Figure A. Treatment should include which of the following?
1. Refrain from weightlifting for a minimum of 6 weeks
2. Physical therapy with rotator cuff strengthening
3. Suprascapular cyst decompression
4. Infraspinatus rotator cuff repair and acromioplasty
5. Spinoglenoid cyst decompression with posterior labral repair Corrent answer: 5
The clinical and MRI findings in the stem are consistent with a posterior labral tear and associated spinoglenoid cyst. A spinoglenoid cyst could cause nerve compression on the suprascapular nerve before its innervation to the infraspinatus, thus causing weakness in external rotation. In contrast, compression at the suprascapular notch would affect both the supraspinatus and infraspinatus muscles. The suggested treatment in a young and competitive athlete with a spinoglenoid cyst would be spinoglenoid cyst decompression with posterior labral repair.
Cummins et al studied suprascapular nerve entrapement and concluded that treatment should be directed toward the underlying cause of the nerve injury. Nonoperative management showed a high rate of failure in the treatment of ganglion cysts. Operative decompression of the suprascapular nerve was associated with a high rate of pain relief and functional improvement. However, resolution of muscle atrophy was less predictable.
Martin et al looked at the results of nonoperative treatment for suprascapular neuropathy confirmed by EMG. Their results suggest that in the absence of a well defined lesion producing mechanical compression of the suprascapular nerve, suprascapular neuropathy could be treated non-operatively.
37) A patient undergoes an MRI arthrogram for recurrent shoulder instability. Based on the imaging, the surgeon feels that arthroscopic treatment is contra-indicated and recommends open treatment. What is the most likely diagnosis?
1. Glenolabral articular disruption (GLAD)
2. Humeral avulsion of the glenohumeral ligament (HAGL)
3. Superior labrum tear from anterior and posterior (SLAP)
4. Anterior labro-ligamentous periosteal sleeve avulsion (ALPSA) 5. Partial articular-sided supraspinatus tendon avulsion (PASTA)
Corrent answer: 2
Humeral avulsion of the glenohumeral ligament (HAGL) occurs when the IGHL tears away from its humeral insertion without an associated subscapular tear. The classic teaching is that HAGL lesions requires open repair of the capsule, whereas the other lesions listed are felt to be better addressed with an arthroscopic approach.
According to the literature review by Stein et al., patients with significant glenoid bone loss, attenuated capsulolabral tissue, engaging Hill-Sachs lesions, and HAGL lesions are contraindicated for arthroscopic repair. They state that while arthroscopy has better cosmesis, decreased perioperative morbidity, decrease loss of external rotation, and is valuable in the confirmation of the extent and severity of shoulder instability, for these lesions open techniques are the gold standard. More recent studies support that arthroscopic treatment of HAGL lesions can still be effective in skilled hands.
The Neviaser article discusses good results for arthroscopic debridement of GLAD lesions for pain relief.
38) A 23-year-old offensive lineman had an arthroscopic anteroinferior labral repair 1 year ago for shoulder instability. He has continued to have recurrent instability. Below is the preoperative MRI from 1 year ago. What is the most likely cause of the recurrent instability?
1. Anteroinferior labral nonunion
2. Unrecognized humeral avulsion of the glenohumeral ligament (HAGL) 3. Anteroinferior glenoid bone defect
4. Engaging Hill Sachs defect
5. Untreated SLAP lesion
Corrent answer: 3
The MRI reveals an anteroinferior glenoid fracture (bony Bankart). This was not addressed at initial surgery as the patient only underwent a soft tissue Bankart repair.
Burkhart et al performed a combined cadaveric and cohort study of patient's without shoulder instability. They examined the use of the glenoid bare spot as a reference point for determining anterior glenoid bone loss. They concluded that the glenoid bare spot is reliably found at the center of the glenoid.
Edwards et al performed a Level 4 review of shoulder instability patients. They found that 73% had the presence of a Hill Sachs lesion and 8% had anterior glenoid bone lesions found on radiographs.
Lynch et al performed a Level 5 review of shoulder instability in the setting of concomitant osseous defects. Soft tissue procedures have a high failure rate when glenoid bony deficiency >25% exists.
39) A 24-year-old bodybuilder reports shoulder pain after an injury while bench pressing. Physical exam reveals ecchymosis and swelling in his right upper arm as shown in Figure A. He has weakness in internal rotation but has good strength in external rotation and
abduction; his apprehension test is negative. When he puts his hands on his hips, his upper chest is asymmetrical. When is surgery indicated for this injury?
1. Surgery is not indicated; conservative management including ice, rest and NSAIDs are recommended
2. After a period of immobilization, followed by physical therapy, has failed 3. When the pectoralis major has avulsed from its humeral insertion 4. Asymmetry of the upper chest wall without functional deficits 5. If swelling and ecchymosis are primarily located on the chest wall rather than the upper arm
Corrent answer: 3
The injury described in this question is consistent with is a humeral avulsion of the pectoralis major muscle, which should be treated with primary surgical repair.
Surgical repair is indicated with complete pecto-humeral tears. Tears are often found on history and examination, however the location of tear often is determined by radiographic adjuncts such as MRI. The standard of practice includes suture anchors or bone tunnels into the pectoralis insertion on the humerus.
Schepsis et al. looked at a retrospective case series of 17 patients with acute and chronic distal pectoralis major muscle rupture. They showed that outcome measures of strength after surgical repair of acute and chronic tears were not significant. However, patients treated operatively fared significantly better than patients treated non-operatively.
Pochini et al. looked at a prospective cohort of 60 patients treated conservatively vs operatively for pectoralis major muscle rupture. They found that the non-operatively group had more complications and decrease in strength of 41.7% relative to 4.3% for the surgical group.
Figure A shows bruising in the right axilla consistent with physical exam findings of a pectorals major tear.
Illustration A shows a T2 weighted MRI with a retracted pectoralis major tendon (straight solid arrow). There is periosteum (curved arrow) pulled off the humeral cortex adjacent to the biceps tendon and hemorrhage within the muscle (open arrow).
Incorrect Answers:
Answer A and B: Operative outcomes show significantly better results than non-operative outcomes.
Answer D: Asymmetry of the chest wall can result from a spectrum of pectorals major tears, including bony tendon avulsions, mid-tendon and musculotendinous tears. Asymmetry of the chest wall alone, does not warrant surgical intervention. Tear location, function and patient factors must also be considered.
Answer E: Swelling and ecchymosis over the chest wall is likely indicative of a complete or incomplete mid-substance pectoralis major tear. These injuries typically do not benefit from surgery.
40) A freshman collegiate swimmer complains of right shoulder pain after increasing his workout duration and intensity. He denies trauma and admits to popping his shoulders in and out voluntarily since the age of 8. Exam reveals bilateral anterior shoulder apprehension and relocation, positive jerk test, and a 2cm sulcus bilaterally. O’Brien active compression tests are negative bilaterally. Radiographs are normal and MR arthrogram of his right shoulder is shown in Figures A and B. What is the best initial treatment?
1. Shoulder range of motion program with emphasis on posterior capsular stretching
2. Shoulder arthroscopy with anterior and posterior capsulolabral plication with superior shift
3. Shoulder arthroscopy with thermal capsulorrhaphy and rotator interval closure
4. Shoulder arthroscopy with repair of humeral avulsion of the glenohumeral ligament (HAGL) lesion
5. Rotator cuff and peri-scapular muscular strengthening program
Corrent answer: 5
This patient has multidirectional shoulder instability a(MDI) and the initial treatment strategy is physical therapy to strengthen the rotator cuff and peri scapular muscles to actively compress the humeral head into the glenoid cavity and support the shoulder girdle. The MRI demonstrates posterior capsular redundancy and an intact anterior and posterior labrum. Any patient with history and exam findings of MDI should initially undergo physical therapy.
The reference by Levine et al. reviews the treatment strategy of MDI and discusses the potential need for operative intervention which include open capsular shifts in non-overhead contact athletes or for revision surgery.
The Robinson et al article reviews posterior instability and the multi-factorial nature and various treatments for this problem.
41) A 51-year-old diabetic female has been treated with non operatively for left shoulder stiffness for the last six months. Despite physical therapy and two corticosteroid injections, she has only been able to achieve 15 degrees of external rotation. She elects arthroscopic treatment. Which of the following interventions would best mitigate the chances of her developing the most common complication of surgical treatment?
1. Perioperative prophylactic intravenous antibiotic administration 2. Avoidance of inadvertent division of the subscapularis tendon 3. Post-operative oral non-steroidal anti-inflammatory drug (NSAID) usage 4. Immediate range of motion and physical therapy
5. Taking care not to divide the inferior capsule further than the thickness of the capsule alone
Corrent answer: 4
This patient has adhesive capsulitis. The primary treatment is nonoperative with a gentle home stretching program, as this is a self-limited process that can take up to 18 months to resolve. If arthroscopic capsular release is attempted, the most common complication is recurrent stiffness.
Infection, instability, impingement syndrome, recurrent stiffness, and axillary nerve injury are all potential complications of arthroscopic capsular release for the treatment of adhesive capsulitis. However, recurrent stiffness is the most
common complication and can be mitigated by immediate early motion and physical therapy with no sling usage. Continuous passive motion machines may be considered as an adjunct. Pain control is important, and regional anesthesia techniques can be helpful in the perioperative period.
Pollock et al. treated 30 shoulders with arthroscopic release for resistant adhesive capsulitis and achieved satisfactory results in 25 (83%). Those with diabetes fared more poorly, with satisfactory results in only 64%.
Yoon et al. randomized 66 patients to receive either subacromial injection, intra-articular injection or hydrodilatation (HD) for treatment of adhesive capsulitis. They found that HD patients had less pain and better function at one month and three months but there were no differences among groups at 6 months. They concluded that HD offered more rapid improvement, but that the three injection techniques offered similar benefits at final follow up.
Sun et al. performed a systematic review and meta-analysis of eight randomized, controlled trials of the effects of intra-articular corticosteroid injection for frozen shoulder. Patients who received injection had less pain and more passive external rotation and abduction at all time points studied (up to 26 weeks). They concluded that steroid injection was a safe and effective treatment for frozen shoulder, resulting in pain relief, functional improvement and increased range of motion at 4-6 weeks, 12-16 weeks and 24-26 weeks post-intervention.
Incorrect Answers
Answer 1: Routine use of prophylactic antibiosis is recommended but infection is not as common as stiffness.
Answers 2 and 5: Iatrogenic injury to nerve or tendon should be carefully avoided but stiffness is more common.
Answer 3: NSAIDs may be useful for controlling post-operative pain and inflammation and therefore secondarily prevent recurrent stiffness but are less critical than motion and therapy.
42) A 24-year-old minor league baseball pitcher presents with shoulder pain. On exam, his strength is normal. At 90 degrees of abduction, he has a total arc of motion of 150 degrees and a loss of internal rotation of 30 degrees. His scapula hangs lower than on the non-throwing shoulder. Initial management should consist of
1. shoulder arthroscopy and SLAP repair
2. shoulder arthroscopy and a capsular release
3. intra-articular cortisone injection, rest and a pitching program 4. diagnostic arthrosopy and subacromial decompression with coracoacromial ligament resection
5. aggressive physical therapy involving posterior capsular stretching and scapular strengthening
Corrent answer: 5
Initial management in throwing athletes with shoulder pain includes rest from throwing with rehabilitaion to improve motion and strength, regardless of the pathology. This patient has glenohumeral internal rotation deficit (GIRD) with
external rotation gain (ERG), which is often seen in asymptomatic throwers. However, the total arc of motion should equal 180 degrees, according to the total motion concept. Burkhart et al discuss the typical Type II SLAP tear seen in pitchers, indicating that posterior capsule tightness increases the risk of this injury. They advocate stretching the posterior capsule and strengthening the entire kinetic chain, including the scapular stabilizers. Braun et al arrive at similar conclusions, and maintain that injections and surgery should be reserved for patients who fail to respond to rest and rehabilitation, with few exceptions.
43) Which of the following is the most common outcome following non-operative management of adhesive capsulitis with a stretching program?
1. Functionally limiting pain
2. Decreased range of motion compared to contralateral shoulder 3. Recurrence of adhesive capsulitis
4. Need for operative intervention
5. Development of rotator cuff arthropathy
Corrent answer: 2
The most common outcome following non-operative management of adhesive capsulitis with a stretching program is decreased range of motion compared to the contralateral side.
Adhesive capsulitis is defined as painful loss of motion of a shoulder without an underlying cause. While it is generally believed to be a self-limiting condition, numerous treatment methods have been suggested including benign neglect,
steroid injections, physical therapy, manipulation, and arthroscopic or open capsular releases. Intra-articular steroid injections may provide an earlier return of shoulder range of motion, but have not shown a long-term difference. Non-operative management with a stretching program shows high rates of patient satisfaction, but it is commonly associated with decreased range of motion compared to the contralateral extremity.
Griggs et al. reviewed 75 patients with phase-2 adhesive capsulitis who were treated non-operatively with a stretching program. At an average follow-up of 22 months, forward flexion increased by 19 degrees, but still remained 36 degrees less than the unaffected shoulder.
Shaffer et al. reviewed 62 patients with adhesive capsulitis who were treated non-operatively with a stretching program. At an average follow-up of 7 years, 60% of patients had decreased range of motion in at least one plane when compared to a control-group of normal shoulders.
Incorrect Answers:
Answer 1: While continued pain is a frequent complication, it is usually much improved from the initial onset of the disease and does not affect quality of life.
Answer 3: Adhesive capsulitis is thought to have a low recurrence rate after it has resolved.
Answer 4: Surgical intervention following non-surgical management of adhesive capsulitis is rare since >90% report satisfaction with non-operative treatment.
Answer 5: The association between rotator cuff arthropathy and adhesive capsulitis has not been studied.
44) A 25-year-old wrestler, who you have been following for 1 year for persistent shoulder pain, presents to your office. He reports that his shoulder pain has not improved despite a period of rest with directed physical therapy and a cortisone injection. On exam, he has full passive and active range of motion. He is tender over the AC joint and has pain but no weakness with resistance while the arm is adducted and flexed forward. Figure A-C show his recent MRI. What is the next best step in treatment?
1. SLAP repair
2. SLAP repair with biceps tenodesis
3. Subpectoral biceps tenodesis
4. Distal clavicle excision
5. Anatomic coracoclavicular ligament reconstruction with distal clavicle excision
Corrent answer: 4
This patient's history and exam findings are classic for distal clavicle osteolysis. Conservative treatment options have been exhausted and he would therefore benefit most from a distal clavicle excision at this time.
Distal clavicle osteolysis is due to repetitive stress and micro-fractures leading to bone resorption. It is most commonly seen in weightlifters who bench press or from similar activities placing undue traction through the AC joint. A trial of rest, NSAIDs, physical therapy and modified weightlifting technique (avoiding
manoeuvres with the elbows posterior to the torso) have shown to be successful in mitigating symptoms. If symptoms persist, open or arthroscopic distal clavicle excision is indicated. The arthroscopic approach has the added benefit of allowing concomitant pathology to be addressed.
Auge et al. reported on a series of 10 arthroscopic distal clavicle excisions performed on weightlifters with distal clavicle osteolysis. All athletes returned to sport at an average of 3.2 days, returned to weight training at 9.1 days, and had increased strength and work volume. This demonstrates the
effectiveness of arthroscopic excision in relieving symptoms in the athlete population.
Roedl et al. reviewed outcomes of distal clavicle osteolysis at a single institution. The mean age was 15.9 years with most participating in overhead sports and/or weightlifting. They found that 93% of patients responded to conservative treatment and that MRIs performed at follow-up showed an average of 5mm of AC joint widening. Their findings support the theory of overhead and weightbearing activities having an important role in the development of distal clavicle osteolysis.
Figures A-C are fluid-sensitive MRI images showing edema within the distal clavicle and AC joint.
Illustration A and B are pre- and post-operative x-rays of a patient after an arthroscopic distal clavicle excision.
Incorrect Answers:
Answers 1 and 3: Though superior labral and biceps pathology is seen in wrestlers, this patient's symptoms and exam are most consistent with distal clavicle osteolysis.
Answer 2: Some degree of weakness would be expected with a rotator cuff tear.
Answer 5: Instability related to AC separation would be the inidication for CC ligament reconstruction, however the history and images are more consistent with distal clavicle osteolysis.
45) A 23-year-old wrestler presents to your office after suffering his fourth anterior shoulder dislocation in one year. A radiograph and MRI are obtained and are shown in Figures A and B, revealing a bony avulsion. Under normal circumstances, in what situation is the ligament that attaches to this avulsed bone subjected to the highest stress?
1. Inferior force in 0 degrees of abduction
2. Anterior force in 90 degrees of abduction and external rotation 3. Anterior force in 45 degrees of abduction and external rotation 4. Posterior force in 90 degrees of forward flexion and internal rotation 5. Posterior force in 45 degrees of abduction and external rotation
Corrent answer: 2
The figures show a humeral avulsion of the glenohumeral ligaments (HAGL) lesion. With the shoulder abducted and externally rotated to 90 degrees, the anterior inferior glenohumeral ligament (aIGHL) is fully taut and vulnerable to injury, sometimes leading to an avulsion from the humeral attachment.
Anterior shoulder instability is a common condition that involves various structures in the shoulder. The anterior band of the inferior glenohumeral ligament is a critical stabilizer with the shoulder in the vulnerable position of 90 degrees of abduction and external rotation. With anterior dislocation or subluxation, a spectrum of injuries to the aIGHL, labrum, bone, and/or cartilage may occur. HAGL lesions usually occur during hyper-abduction with external rotation events and are best seen on MRI. Anatomic variations have been described including humeral bony avulsions and ligamentous avulsions (bony HAGL) with concomitant capsulolabral detachment (floating anterior IGHL).
Bui-Mansfield et al. reported a case series of HAGL lesions. All cases involved males who had undergone a large traumatic event. They noted that all HAGL lesions were found in conjunction with other pathology both intra-articular (long head of biceps tear, Bankart lesion, humeral OCD) and extra-articular (RTC tears and clavicle fractures).
George et al. reviewed the management of HAGL lesion. They reported more severe anterior shoulder pain in cases of HAGL lesions and recommend a high index of suspicion for an unrecognized HAGL lesion in cases of persistent instability. Arthroscopic and open techniques are both described with successful results, however, it may be more challenging arthroscopically as it often requires the use of a 70-degree arthroscope and uncommon accessory portals.
Figure A is a radiograph showing a bony HAGL lesion. Figure B is a T2 coronal MR image showing the classic appearance of a HAGL lesion. Illustration A is a radiograph with a more apparent bony HAGL lesion (Bui-Mansfield et al). Illustration B shows an MR image with the curved arrow labeling the J-shaped IGHL and black arrow noting the fluid extravasation (Bui-Mansfield et al). Illustration C depicts the glenohumeral ligaments (George et al)
Incorrect Answers:
Answer 1: The SGHL is taut at 0 degrees of abduction and resists inferiorly directed forces
Answers 3 and 5: With the shoulder at 45 degrees of abduction the MGHL is taut and resists both anteriorly and posteriorly directed forces
Answer 4: The posterior IGHL is taut in 90 degrees of abduction with external rotation and resists posteriorly directed forces
46) A 24-year-old rugby player presents to you for recurrent shoulder injuries. He has tried physical therapy which has not helped and he could not tolerate bracing. Based on his imaging is shown in Figures A-D, you recommend surgery. What procedure(s) is indicated and what additional restrictions would this require post-operatively?
1. Rotator cuff repair, avoid internal rotation and active abduction 2. Latarjet procedure, avoid external rotation and abduction 3. Arthroscopic bankart repair and remplissage, avoid abduction in the scapular plane
4. Arthroscopic bankart repair and remplissage, avoid forward flexion and adduction
5. Latarjet procedure, avoid resisted elbow flexion and supination Corrent answer: 4
The imaging shows a large anterior labral Bankart lesion and a Hill-Sachs lesion. The most likely procedure needed to be performed in conjunction is a remplissage procedure. This requires avoiding the vulnerable position of adduction and forward flexion (analogous to the Jerk test in the posterior instability setting). Forward flexion and adduction tensions the posterior capsular structures and would jeopardize the repair.
Anterior shoulder instability is a common occurrence among collision sports, with rugby and American football being the most common. Recurrence rates are indirectly correlated to age, with teenage athletes seeing rates of approximating 90%. In-season bracing is effective in limiting further dislocations until post-season surgery to address the Bankart lesion can be performed. Associated lesions seen in anterior instability are Hill-Sachs defects, HAGL lesions, and GLAD lesions. Hill-Sachs are most often treated with remplissage procedure if they are "off-track" and engaging. Remplissage involves placing the infraspinatus tendon and posterior capsule into the defect rendering it extra-articular.
Zhu et al. reported a series of patients undergoing anterior stabilization with arthroscopic bankart repair and remplissage. Forty-nine patients underwent this procedure with clinically significant increases in Rose, ASES and Constant scores at an average follow-up of 29 months. Decreases in internal rotation were recorded at under 2 degrees, showing this method as a feasible alternative to prior techniques.
Parnes et al. reported their technique of arthroscopic remplissage. They used a transtendinous suture anchor repair, where limbs from both anchors were tied together and pulled down in a pulley fashion. This can be visualized either intra-articularly or subacromially and has the advantage of placing a broad portion of infraspinatus/capsule into the defect. Post-operative external rotation immobilizer is used briefly to limit internal rotation or forward flexion and protect the posterior capsular repair.
Figure A is a normal AP radiograph of the right shoulder. Figures B-D are fluid sensitive MR arthrogram images showing a large anterior Bankart lesion without bone loss and a Hill-Sachs lesion. There is no fatty atrophy of the rotator cuff muscles. Illustration A is from Zhu et al. and demonstrates the remplissage procedure. Illustrations B-D are from Parnes et al. showing an arthroscopic remplissage with a pulley method.
Incorrect Answers:
Answer 1- Rotator cuff tears may occur after dislocations, but the imaging shows intact rotator cuff tendons and no muscle atrophy.
Answers 2 and 5- Latarjet is done to augment large anterior bony defects, which this patient does not have. Additionally, elbow flexion and supination should be incorporated in early rehab to avoid elbow stiffness. Answer 3- Adduction, not abduction should be avoided in post-op rehab after remplissage.
47) A 58-year-old right-hand-dominant firefighter presents with a chief complaint of chronic right elbow pain. He denies prior trauma. He has previously tried non-steroidal anti-inflammatories, physical therapy, and corticosteroid injections without significant relief. On physical exam, he is well-built without gross deformity, his active and passive range of motion is 30-130 (extension-flexion) degrees, pronation is 70 degrees, and supination is 85 degrees. He exhibits crepitus on exam and has some tenderness at end-range of motion. Radiographs are obtained and are presented in Figures A and B. What
is the most appropriate treatment option at this time with the most predictable pain relief?
1. Repeat corticosteroid injection and physical therapy
2. Static splinting
3. Arthroscopic debridement
4. Interposition arthroplasty
5. Open debridement
Corrent answer: 3
In a younger, high-demand patient with moderate elbow osteoarthritis failing non-operative management, the next best treatment of choice would entail arthroscopic debridement. Arthroscopic debridement provides more reliable pain relief than open debridement, which provides more predictable improvements in range of motion. This patient has an acceptable motion arc and his primary complaint is pain.
Symptomatic primary elbow arthritis is rare and primarily affects middle-aged male laborers. The radiocapitellar joint is primarily affected, and osteophytes, capsular contracture, and loose bodies are the main contributors to pain and functional impairments. Initial treatment is non-operative, with a trial of non steroidal anti-inflammatories, resting splints, activity modification, and physical therapy. Operative treatment depends on age, activity demands, degree of arthritis, and specific pathology. Major indications for elbow arthroscopy include irrigation and debridement in the setting of septic arthritis, synovectomy for inflammatory arthritis, debridement and loose body removal for osteoarthritis, contracture release, treatment of osteochondral defects, and lateral epicondyle release in the setting of tennis elbow. With more advanced radiocapitellar arthritis, radial head replacement is a consideration. Total elbow arthroplasty is not advised in younger, high-demand patients, as there is a lifetime 5-10 pound lifting restriction following this procedure.
Pitta et al. provide a review article on the arthroscopic management of osteoarthritis in general. They comment on the controversial role of arthroscopy in the setting of osteoarthritis and conclude that arthroscopy may still serve as an effective and less-invasive option in certain clinical situations. Elbow arthroscopy for osteoarthritis often involves the removal of loose bodies, osteophyte resection, and capsular release. Arthroscopy appears to more reliably relieve pain than improve motion.
Sears et al. discuss the evaluation and management of post-traumatic elbow arthritis in a younger demographic. They note that, due to high functional demands, total elbow arthroplasty is a salvage procedure in this cohort; advanced cases of elbow osteoarthritis are often better addressed with resurfacing procedures or interposition arthroplasty. They conclude that the goal of surgery in this group is to improve pain and functional motion in a
conservative manner as to not exclude surgical options down the road.
Figures A and B are anteroposterior and lateral radiographs of a right elbow, respectively, demonstrating moderate osteoarthritis. The joint spaces are relatively well preserved in the anteroposterior view. Osteophytes are identified at the anterior humerus, coronoid tip, and olecranon. Illustrations A and B are axial and sagittal computed tomographic scans of the elbow demonstrating osteophyte formation in the olecranon and coronoid fossae, as well as osteophytes at the coronoid and olecranon tips.
Incorrect Answers:
Answer 1: Repeating corticosteroid injections and physical therapy are unlikely to benefit this patient, as he has failed these non-operative modalities previously.
Answer 2: Static splinting is a treatment option for contractures about the elbow, however, this patient has a functional arc of motion and his chief complaint is pain.
Answer 4: Interposition arthroplasty may be a treatment option in a younger, high-demand patient who would otherwise be a candidate for a total elbow arthroplasty.
Answer 5: Open debridement is more reliable at improving range of motion, whereas arthroscopic debridement is more reliable at relieving pain. This patient has adequate motion and his chief complaint is pain.
48) A 67-year-old male presents with left shoulder pain. He
underwent surgery on his left shoulder 10 years prior. Initially, he was pain-free following surgery, however, his pain has returned and has been increasing in intensity. On physical examination, his incision is
healed with no erythema. He actively exhibits 120° of forward flexion, 25° of external rotation, and internal rotation to L3. He exhibits 5/5 strength with forward flexion, internal rotation, and external rotation. He has a negative belly-press test, negative hornblower's sign, and a negative Spurling's test. Distally, he is neurovascularly intact. Joint aspiration is performed in the office and reveals a white blood cell count (WBC) of 1900 x10^9/L, with 20% polymorphonuclear leukocytes (PMNs). Cultures were held for 3 weeks and exhibited no growth to date. Radiographs are obtained and shown in Figures A & B. Which of the following is the next best step in the treatment of this patient?
1. Magnetic resonance imaging (MRI) of the shoulder to evaluate the rotator cuff
2. Humeral head revision and placement of prosthetic glenoid component 3. Two-stage revision with placement of an antibiotic spacer 4. Revision total shoulder arthroplasty with a cemented humeral stem 5. Revision to an implant with a center of rotation that is moved infero medially
Corrent answer: 2
Figures A and B demonstrate a left shoulder hemiarthroplasty with associated glenoid erosion. The next best step in this patients treatment is a revision of the humeral head and placement of a prosthetic glenoid component.
Shoulder hemiarthroplasty (HA) is currently used to treat glenohumeral arthritis and treatment for proximal humerus fractures. It has been increasingly utilized in younger active patients with glenohumeral arthritis due to the risk of glenoid loosening. Some patients treated with HA will continue to complain of pain which may be associated with glenoid arthritis and wear. For patients who experience this, a revision to total shoulder arthroplasty (TSA) is the treatment of choice. With the advent of modular systems, the humeral stem may be retained with the placement of a prosthetic glenoid component.
Groh et al. performed a study to determine the difficulty and results of revision from hemiarthroplasty to total shoulder arthroplasty utilizing modular systems. 15 patients who underwent revision from hemiarthroplasty to total shoulder arthroplasty were reviewed. They found that revision of HA to TSA was associated with pain relief as well as improvement in forward elevation. They conclude that the revision of painful HA for glenoid arthrosis to TSA is a reliable procedure.
Carroll et al. evaluated the outcomes of patients after conversion of painful HA to TSA. They found 3 excellent results, 5 satisfactory results, and unsatisfactory results in 7. 7/11 patients had posterior glenoid erosion. They conclude that revision of a failed HA to a TSA is a salvage procedure whose results are inferior to those of primary TSA.
Figure A is an AP radiograph of the shoulder demonstrating a well-fixed hemiarthroplasty with glenoid erosion. Figure B is an axillary radiograph demonstrating the same.
Illustrations A & B are AP and axillary radiographs, respectively, demonstrating conversion of the hemiarthroplasty to a total shoulder arthroplasty.
Incorrect Answers:
Answer 1: MRI is not indicated to evaluate the rotator cuff as the patient's rotator cuff appears to be intact from the physical examination. Visualization of the rotator cuff would also be difficult given the presence of a hemiarthroplasty.
Answer 3: Two-stage revision with an antibiotic spacer is unnecessary as this patient does not exhibit signs of infection, and cultures were negative at 3 weeks.
Answer 4: Explantation and revision to a total shoulder arthroplasty would not be appropriate, as removal of the current, well-fixed humeral stem would add
significant morbidity to the procedure.
Answer 5: Revision to a reverse total shoulder arthroplasty at this time would not be appropriate, as the patient has a functioning rotator cuff and there is no proximal migration of the stem.
49) A 20-year-old male right hand dominant college baseball pitcher presents with 6 months of vague right shoulder pain. He reports decreased performance and pitch velocity as a result of this ongoing problem. On physical exam, he demonstrates the findings shown in Figure A. Which of the following is the most appropriate initial treatment strategy for this patient?
1. Arthroscopic posteroinferior capsule release
2. Arthroscopic anteriorinferior capsule release
3. Posterior labral repair with capsulorrhaphy
4. Physical therapy directed at anteriorinferior capsule stretching 5. Physical therapy directed at posteroinferior capsule stretching
Corrent answer: 5
This patient has a finding of glenohumeral internal rotation deficit (GIRD), which can cause shoulder and elbow problems. Internal impingement of the shoulder is one of the most commonly associated syndromes, which is best treated with a course of physical therapy directed at posteroinferior capsule stretching.
Glenohumeral internal rotation deficit (GIRD) occurs most commonly in overhead throwing athletes and characteristically demonstrates a posterior shift in the glenohumeral range of motion (increased external rotation and decreased internal rotation). Loss of internal rotation of >25 degrees is felt to be clinically relevant GIRD. The first line of treatment consists of rest from throwing and therapy to address rotation deficits. One of the most common stretches is the sleeper stretch, where the patient provides internal rotation to the shoulder with the arm in 90 degrees of abduction. Rarely, patients who fail physical therapy and stretching may be indicated for arthroscopic posterior
capsule release.
Braun et al. reviewed shoulder injuries in throwing athletes. They discuss treatment for GIRD, stating the majority of patients respond to physical therapy. Those few who fail physical therapy are often older, elite athletes and can be treated with arthroscopic posteroinferior capsulotomy in the posterior band of the inferior glenohumeral ligament.
Crockett et al. looked at 25 professional throwing athletes and 25 non throwing patients for range of motion as well as humeral head and glenoid retroversion. They found the dominant shoulder of throwing athletes had significantly greater humeral head and glenoid retroversion as well as external rotation when compared to the non-dominant shoulder. They found no difference in range or motion or retroversion in the dominant versus non dominant shoulders of non-throwing patients. They concluded that the dominant shoulders of throwing athletes have greater retroversion and internal rotation.
Figure A shows a patient with loss of internal rotation on the right side. Illustration A shows a patient performing a "sleeper stretch", which stretches the posterior capsule in GIRD.
Incorrect Answers:
Answer 1: Arthroscopic posteriorinferior capsule release would only be indicated in rare cases when physical therapy fails to improve the patient's symptoms and range of motion.
Answer 2: Arthroscopic anterior inferior capsule release is not indicated in cases of GIRD
Answer 3: Posterior labral repair with capsulorrhaphy would be indicated in cases of posterior instability more often seen in lineman or weightlifters. Answer 4: The anterior inferior capsule has increased length in GIRD and would not require additional stretching.
50) A 52-year-old female with adhesive capsulitis undergoes an arthroscopic release after failure of nonoperative management. Releasing which of the following labeled structures should result in improved cross-body adduction?
1. A
2. C
3. H
4. E & C
5. B & D
Corrent answer: 3
Release of the posterior capsule (label H) would result in improved cross-body adduction (adduction of the forward-flexed shoulder) intra-operatively.
Adhesive capsulitis (frozen shoulder) is a fibroblastic proliferation of capsular
tissue of unknown etiology resulting in pain and loss of both passive and active shoulder range of motion. While nonoperative management is successful in the majority of patients, surgery may be considered in recalcitrant cases. Classically, the rotator interval is released to improve external rotation, while the posterior capsule is released to improve internal rotation and cross-body adduction.
Kim et al. performed a randomized controlled trial of 75 patients with shoulder stiffness to compare outcomes in patients with and without posterior extended capsular release. Despite intra-operative improvements, at final follow-up, the authors found no difference in forward flexion, external rotation, and internal
rotation between the groups. They concluded that posterior extended capsular release may not be necessary in the arthroscopic management of shoulder stiffness.
Snow et al. evaluated the efficacy of posterior arthroscopic release in 48 patients with frozen shoulder. The authors found that, compared to the group that underwent only an anterior and inferior release, patients who also had a posterior release had no difference in Constant Scores and range of motion, particularly internal rotation. They concluded there was no significant difference in outcome with the addition of a posterior release in patients with a frozen shoulder.
Illustration A shows the capsular and ligamentous structures important for glenohumeral stability.
Incorrect Answers:
Answer 1: A shows the long head of the biceps. While its role as a pain generator in various shoulder pathologies has been debated, release of the long head of the biceps would not increase cross-body adduction. Answer 2: C shows the middle glenohumeral ligament (MGHL). The MGHL resists anterior and posterior translation in ~45 degrees of abduction. Answer 4: E shows the anterior band of the inferior glenohumeral ligament (AB-IGHL), while C shows the MGHL. The AB-IGHL is the primary restraint to anterior and inferior translation with the arm at 90 degrees abduction and maximum external rotation.
Answer 5: B shows the superior glenohumeral ligament (SGHL), while D shows the anterior capsule. The SGHL is a restraint to inferior translation at 0 degrees of abduction.
51) An active 61-year-old male sustains the injury seen in Figure A. After undergoing treatment for this acute condition he continues to have pain and weakness in his shoulder for 6 weeks. Which of the following is the next best step?
1. Magnetic resonance imaging (MRI) of the shoulder to evaluate the rotator cuff
2. Magnetic resonance imaging (MRI) of the shoulder to evaluate the labrum 3. Electromyography to evaluate for Axillary Nerve Palsy
4. Computed tomography (CT) of the the shoulder to evaluate the glenoid 5. Computed tomography (CT) of the shoulder to assess for an occult fracture
Corrent answer: 1
This patient sustained an anterior shoulder dislocation. Following shoulder dislocations, elderly patients should be evaluated for rotator cuff tears with MRI.
Rotator cuff tears following shoulder dislocations have been found to occur in
up to 80% of patients over the age of 60. Persistent pain and dysfunction after a shoulder dislocation are signs that a rotator cuff tear may be present and an MRI should be obtained. After a trial of physical therapy, untreated tears may cause persistent pain, dysfunction, instability, and degenerative changes. Surgical treatment of these injuries usually leads to pain relief and restoration of function.
Gombera et al. performed a systematic review to determine when a rotator cuff tear should be suspected after a dislocation, whether surgical or nonsurgical approaches result in better scores for pain and satisfaction in patients with rotator cuff tears resulting from shoulder dislocations, and whether intraarticular lesions, rotator cuff tears, or both should be addressed when surgery is performed. They found that patients with persistent pain or dysfunction often had a concomitant rotator cuff tear and that surgical repair resulted in improved pain relief and higher patient satisfaction compared to non-operative management.
Wolf et al. performed a review on the indications for repair of full-thickness rotator cuff tears. They report that acute tears are reported to make up 8% of all rotator cuff tears and are usually associated with a traumatic event such as a fall or shoulder dislocation. They report MRI is useful in these situations to evaluate for fatty degeneration, atrophy, and retraction.
Figure A is an AP radiograph of the shoulder demonstrating medial displacement of the humeral head relative to the glenoid fossa. Figure B is an axillary radiograph of the shoulder that shows the humeral head dislocated anteriorly and impacted on the anterior aspect of the glenoid. Illustration A is a T2-weighted coronal MRI slice demonstrating a full-thickness rotator cuff tear indicated by an arrow. The asterisk delineates distention of the AC joint capsule. Illustration B is a photograph demonstrating a full-thickness rotator cuff tear.
Incorrect Answers:
Answer 2: MRI of the shoulder should be obtained to evaluate the rotator cuff, not the labrum.
Answer 3: Electromyography would not be the next best test to obtain at this time. Rotator cuff tear is more likely than axillary nerve injury in a patient over 40.
Answer 4: A CT to evaluate the glenoid is not initially indicated in an elderly patient following a shoulder dislocation. This may be obtained at a later time if arthroplasty is being planned to assess version of the glenoid. Answer 5: A CT to evaluate for an occult fracture is not indicated in this scenario.
52) A 55-year-old right-handed female presents with left shoulder pain and stiffness. She denies any medical problems. On physical examination, her range of motion is notable for passive and active forward flexion to 100°, abduction to 40°, external rotation to 5° and internal rotation to L5. Imaging of the left shoulder is shown in Figures A and B. Which of the following is most likely to be present in this patient?
1. Elevated thyroid stimulating hormone (TSH)
2. Increased serum uric acid
3. Elevated erythrocyte sedimentation rate (ESR)
4. Positive anti-cyclic citrullinated peptide (anti-CCP)
5. A history of multiple shoulder dislocations
Corrent answer: 1
This patient has adhesive capsulitis, which has been associated with hypothyroidism, which would show elevated thyroid stimulating hormone (TSH).
Adhesive capsulitis (AC), or frozen shoulder, is a condition of uncertain etiology characterized by significant restriction of both active and passive shoulder motion that occurs in the absence of a known intrinsic shoulder disorder. It is most commonly seen in women 40-60 years of age. Endocrine, immunological, and inflammatory etiologies have been proposed. Thyroid dysfunction, diabetes mellitus, Dupuytren contractures, breast cancer treatment, and autoimmune diseases have been historically associated with AC. A complete workup in patients with newly diagnosed AC should be performed prior to selecting a treatment option.
Schiefer et al. performed a case-control study to compare patients with AC to controls. A specific questionnaire was applied, and measurements of serum thyroid-stimulating hormone (TSH) and free tetraiodothyronine were performed in all subjects. They found that hypothyroidism was significantly higher in patients with AC. Also, patients with elevated TSH levels had a higher prevalence of bilateral AC. They conclude that hypothyroidism was significantly higher in AC patients than in controls.
Huang et al. performed a study to determine the prevalence and adjusted hazard ratio (HR) of AC among patients who have hyperthyroidism. They obtained data from a database in Taiwan and records were reviewed. Of 4472 patients with hyperthyroidism, 162 experienced AC (incidence 3.6%). They found the adjusted HR of developing AC was 1.22 which was statistically significant. They conclude that hyperthyroidism is an independent risk factor of developing adhesive capsulitis.
Figure A is a normal AP radiograph of the shoulder. Figure B is a T2 weighted MRI demonstrating increased signal of the joint capsule in the axillary recess.
Incorrect Answers:
Answer 2: An increased serum uric acid would be consistent with gout. Answer 3: An elevated ESR would be consistent with infection.
Answer 4: A positive anti-CCP would be consistent with rheumatoid arthritis. Answer 5: A history of previous shoulder dislocation is not associated with AC.
53) An 18-year-old male presents to the emergency department with left shoulder pain following a seizure earlier in the day. He reports a history of epilepsy and that he has dislocated his left shoulder over ten times. He is inconsistent with taking his seizure medications. A radiograph of his shoulder is shown in Figure A. Upon reviewing his electronic medical record a CT scan of his left shoulder from one month ago is found as demonstrated in Figure B. Which of the following treatment options is the most appropriate at this time?
1. Do not attempt closed reduction and admit for closed versus open reduction in the operating room
2. Closed reduction alone
3. Closed reduction and referral to neurology
4. Closed reduction and outpatient referral for shoulder stabilization procedure 5. Closed reduction and outpatient referral for shoulder fusion
Corrent answer: 3
In patients with epilepsy and shoulder instability, surgery should not be performed until medical management is exhausted. This patient should be closed reduced, referred to neurology, and, once his seizures are under control, considered for shoulder stabilization surgery; bone grafting for his glenoid deficit (>20-25%) would be indicated.
Large bony defects and degenerative changes are the characteristic findings in epileptic patients with recurrent shoulder dislocations. Muscle contractions and trauma are the main causes of shoulder dislocation during seizure activity. Soft tissue stabilization procedures alone are often unsuccessful, as it is imperative to address the bone loss contributing to recurrent instability in this population. Coracoid transfer (Laterjet) and bone grafting can be successful in these patients when seizures are adequately controlled and patient compliance is optimal.
Raiss et al. reported on the results of the Laterjet procedure for epileptic patients in the setting of recurrent anterior shoulder dislocations. In their cohort of 12 patients, they reported an unacceptably high failure rate. They concluded that the Laterjet procedure is contraindicated in patients with uncontrolled seizure activity.
Buhler et al. looked at 34 shoulders in 26 epileptic patients over a 10 year period to examine their pattern of instability and structural lesions. They found an equal rate of anterior and posterior dislocations, characteristically associated with large bony lesions (Hill-Sachs and bony Bankart). They concluded that anterior dislocations are more difficult to treat than posterior dislocations in this group and that skeletal reconstruction is necessary to obtain clinical stability.
Figure A is a left shoulder radiograph demonstrating an anterior dislocation. Figure B is a left shoulder CT demonstrating significant anterior glenoid bone loss.
Incorrect Answers:
Answer 1: There is no reason why a closed reduction attempt should not be attempted in the emergency department.
Answer 2: A closed reduction alone will not prevent this adolescent from
dislocating in the future and does not address his seizures.
Answer 4: Shoulder stabilization surgery should not be performed until medical management of his seizures is exhausted, as this is associated with a high failure rate.
Answer 5: Shoulder fusion is not appropriate at this time.
54) A 25-year-old male baseball player presents 3 months post operatively after undergoing repair of the condition shown in Figure A. He is complaining of shoulder pain and weakness. As the operative surgeon, you recall errantly passing instrumentation medial to the glenoid. From which portion of the brachial plexus does the injured nerve arise?
1. Superior trunk
2. Middle trunk
3. Roots of C5-7
4. Posterior Cord
5. Medial Cord
Corrent answer: 1
The suprascapular nerve (SSN) arises from the superior trunk of the brachial plexus and may be injured during superior labrum anterior-to-posteror (SLAP) tear repair if instrumentation is passed medial to the glenoid.
Neuropathy of the SSN is frequently caused by space-occupying lesions. The nerve may also be injured by plating of the glenoid or muscle mobilization during cuff repair. Injuries have also been reported with SLAP repairs, especially if anchors or instrumentation are passed medial to the glenoid.
Cummins et al. performed an anatomic study on the spinoglenoid ligament. They dissected 112 shoulders and found no distinct ligament structure in 20% of shoulders. Overall, 80% of shoulders had a fibrous band of tissue that, together with the spine of the scapula, formed a narrow fibro-osseous tunnel through which the suprascapular nerve traveled. They found the ligaments to be composed of collagen on histologic analysis. They conclude that the ligament may limit mobilization and advancement of the infraspinatus tendon during repair of a massive cuff tear and the spinoglenoid ligament represents a potential site for nerve entrapment.
Kim et al. present a case of a 41-year-old woman who underwent arthroscopic repair of a SLAP lesion with suture anchors. The patient continued to have pain post-operative and MRI revealed that the suture anchor was beyond the cortical bone of the glenoid rim and abutted the SSN. Direct compression of the nerve was visualized with repeat arthroscopy and the suture anchor was removed. The patient became asymptomatic 3 months later. The authors recommend careful placement of suture anchors during repair of SLAP lesions to avoid iatrogenic injury to the SSN.
Scully et al. perform a review on iatrogenic nerve injuries in shoulder surgery. They report that due to the intimate relationship of the SSN to the glenoid rim as it traverses the body of the scapula, the nerve is at risk during procedures that involve drilling and passing sutures and/or placing screws around the glenoid face and rim. They report that repair of SLAP tears can result in injury as a result of "drill out," in which the drill perforates the medial glenoid and improper anchor placement.
Figure A is T2-weighted MRI slice demonstrating a SLAP tear. Illustration A is a T-1-weighted MRI slice demonstrating suture anchors medial to the glenoid (Kim et al.). Illustration B demonstrates the course of the suprascapular nerve on the posterior scapula with the safe zone depicted with grey shading (Scully et al.) Illustration C demonstrates the safe zones of the glenoid in the transverse plane while Illustration D demonstrates the safe zone in the sagittal plane (Scully et al.).
Incorrect Answers:
Answer 2: There are no branches arising from the medial trunk. Answer 3: The long thoracic nerve arises from the nerve roots of C5-7. Answers 4 & 5: Many nerves arise from the posterior and medial cords of the brachial plexus, and none of them would be injured by passing instrumentation medial to the glenoid.
55) Untreated progression of the condition shown in Figure A may lead to which of the following radiographic appearances shown in Figures B-F?
1. Figure B
2. Figure C
3. Figure D
4. Figure E
5. Figure F
Corrent answer: 4
A rotator cuff tear can lead to rotator cuff tear arthropathy, characterized radiographically by degenerative changes of the glenohumeral joint and superior migration of the humeral head.
Rotator cuff tear arthropathy is a term first used by Charles Neer in 1983. While it is a broad term, all patients with cuff tear arthropathy have at least 3 features in common: a non-functioning rotator cuff, degenerative changes of the glenohumeral joint, and superior migration of the humeral head. The accurate diagnosis of rotator cuff tear arthropathy is important, because definitive treatment changes according to whether a patient has a functioning rotator cuff.
Nam et al. reviewed rotator cuff tear arthropathy. They noted that cuff tear arthropathy always involves the three features of rotator cuff insufficiency, degenerative changes in the glenohumeral joint, and superior humeral head migration. They also pointed out that while there are many patients with
rotator cuff pathology, not all develop rotator cuff arthropathy.
Feeley et al. also reviewed cuff tear arthropathy. The authors stated that, while cuff tear arthropathy has been recognized for decades, a treatment strategy with uniformly satisfactory outcomes remains elusive. They noted that the use of reverse total shoulder arthroplasty to treat this condition showed good results, and is now the standard.
Figure A shows a complete rotator cuff tear of the supraspinatus and infraspinatus. Figure B shows a chondrosarcoma of the proximal humerus. Figure C shows glenohumeral arthritis without rotator cuff arthropathy. Figure D shows a shoulder dislocation. Figure E shows rotator cuff arthropathy of the glenohumeral joint with superior migration of the humeral head and acetabularization of the acromion. Figure F shows a proximal humerus fracture.
Incorrect Answers:
Answers 1 (chondrosarcoma), 2 (glenohumeral osteoarthritis), 3 (dislocation), and 5 (fracture) all show conditions that do not result from the untreated progression of rotator cuff pathology.
56) A 76-year-old female presents with right shoulder pain and inability to actively forward flex her shoulder past 45 degrees. She has had shoulder pain for years but has had no surgical interventions and comes to discuss shoulder replacement. She has a history of hypertension, diabetes, and is a current smoker. Radiographs are shown in Figures A and B. Which of the following should be performed next in preparation for shoulder arthroplasty?
1. Smoking cessation program
2. A trial of physical therapy
3. Prescribe tramadol
4. Lab tests (CBC, ESR, CRP)
5. Referal to a rheumatologist
Corrent answer: 1
A smoking cessation program should be instituted as smoking has been suggested as a contraindication to reverse and total shoulder arthroplasty (RSA,TSA).
RSA and TSA have been associated with complications including infections, loosening, and fractures. Patient-specific factors that have been demonstrated to increase these complications include a younger age, higher body mass index, rheumatoid arthritis, and diabetes. Specifically, smoking has been found to contribute to higher infection rates and increased risks of fracture and loosening. Smokers who are planning to undergo elective, primary shoulder arthroplasty should be enrolled in smoking cessation programs to decrease their risks for developing a complication.
Hatta et al. performed a retrospective study to examine the effect of smoking on the incidence of complications after TSA and RSA. Current smokers, former smokers, and nonsmokers were compared for periprosthetic infection, fractures, and loosening after surgery. There were 20 periprosthetic infections (16 in smokers and 4 in nonsmokers), 27 periprosthetic fractures (14 in smokers and 13 in nonsmokers), and 28 loosenings (14 in smokers and 14 in nonsmokers). Current and former smokers also had significantly higher risks of periprosthetic infections when compared to nonsmokers (hazard ratio of 7.27 and 4.56, respectively). They conclude that smoking is a significant risk factor for complications after TSA and RSA.
Cheung et al. performed a review of complications in RSA. They report that infection is a relatively common complication in RSA with an incidence of 1- 10%. They note that patients who present with rheumatoid arthritis are at increased risk of infection. They conclude that regardless of etiology, management of infection is similar to that following other total joint arthroplasty procedures.
Figure A is an AP radiograph of the shoulder demonstrating proximal humeral migration and cuff-tear arthropathy. Figure B is a scapular Y view of the shoulder demonstrating the same.
Incorrect Answers:
Answer 2: A trial of physical therapy is not likely to benefit this patient as she has advanced cuff tear arthropathy and pseudoparalysis.
Answer 3: Tramadol is recommended as first-line treatment of osteoarthritis of the knee.
Answer 4: CBC, ESR, and CRP should be ordered if one is considering the diagnosis of infection. The likelihood of infection in this scenario is low. Answer 5: Referral to a rheumatologist should be performed if there is clinical suspicion for an unknown diagnosis of rheumatoid arthritis. In this case, the imaging findings are diagnostic and a smoking cessation program should be instituted prior to arthroplasty.
NEXT:ORTHOPEDIC MCQS OB 20 SHOULDER AND ELBOW 2
FOR ALL MCQS CLICK THE LINK ORTHO MCQ BANKFOR ALL MCQS CLICK THE LINK ORTHO MCQ BANK